Đến nội dung

le_hoang1995

le_hoang1995

Đăng ký: 29-06-2011
Offline Đăng nhập: 09-01-2016 - 21:52
*****

#544603 CMR: $\dfrac{ab}{a+b-c}+\dfrac{bc...

Gửi bởi le_hoang1995 trong 16-02-2015 - 23:02

Cho a,b,c là 3 cạnh của tam giác.CMR: $\dfrac{ab}{a+b-c}+\dfrac{bc}{b+c-a}+\dfrac{ca}{a+c-b}\geq a+b+c$

BĐT tương đương với

$\sum \left ( \frac{ab}{a+b-c}-c \right )\geq 0$

$\Leftrightarrow \sum \frac{ab-bc-ac+c^2}{a+b-c}\geq 0$
$\Leftrightarrow \sum \frac{1}{a+b-c}.(a-c)(b-c)\geq 0$
Đặt được ứng, BĐT tương đương $S_c.(a-c)(b-c)+S_a.(c-a)(b-a)+S_b.(a-b)(c-b)\geq 0$

Không mất tổng quát giả sử $a\geq b\geq c\Rightarrow S_a\geq S_b\geq S_c>0$
Suy ra $S_c.(a-c)(b-c)\geq 0$
Và $S_a.(a-c)\geq S_b.(a-c)\geq S_b.(b-c)$

$\Rightarrow S_a.(a-c).(a-b)\geq S_b.(b-c)(a-b)\Leftrightarrow S_a.(c-a)(b-a)+S_b.(a-b)(c-b)\geq 0$

$\Rightarrow S_c.(a-c)(b-c)+S_a.(c-a)(b-a)+S_b.(a-b)(c-b)\geq 0$

BĐT đã được chứng minh. Đẳng thức xảy ra khi và chỉ khi $a=b=c$




#542212 $\frac{x}{x+\sqrt{x+yz}}+\f...

Gửi bởi le_hoang1995 trong 28-01-2015 - 22:40

Với 3 số dương $x,y,z$ và $x+y+z=1$.Tìm max

$\frac{x}{x+\sqrt{x+yz}}+\frac{y}{y+\sqrt{y+xz}}+\frac{z}{z+\sqrt{z+xy}}$

Theo BĐT cauchy-schwarz ta có

$x+\sqrt{x+yz}=x+\sqrt{x^2+xy+xz+yz}=x+\sqrt{(x+y)(z+x)} \geq x+\sqrt{xy}+\sqrt{xz}=\sqrt{x}.(\sqrt{x}+\sqrt{y}+\sqrt{z})$

Vậy $VT=\sum \frac{x}{x+\sqrt{x+yz}}\leq \sum \frac{\sqrt{x}}{\sqrt{x}+\sqrt{y}+\sqrt{z}}=1$

Đẳng thức xảy ra khi và chỉ khi $x=y=z=\frac{1}{3}$




#536442 $\sum \frac{a^{2}}{a+b}+\fr...

Gửi bởi le_hoang1995 trong 06-12-2014 - 20:17

Cho a, b, c > 0. Chứng minh $\sum \frac{a^{2}}{a+b}+\frac{1}{2}\sum \sqrt{ab}\geqslant \sum a$

BĐT cần chứng minh tương đương với:

$\sum \left ( a-\frac{ab}{a+b} \right )+\frac{\sqrt{ab}+\sqrt{bc}+\sqrt{ca}}{2}\geq a+b+c$
$\Leftrightarrow \frac{\sqrt{ab}+\sqrt{bc}+\sqrt{ca}}{2}\geq \frac{ab}{a+b}+\frac{bc}{b+c}+\frac{ca}{c+a}$

Nhưng BĐT này đúng do $\frac{\sqrt{ab}}{2}=\frac{ab}{2\sqrt{ab}}\geq \frac{ab}{a+b}$

Đẳng thức xảy ra khi $a=b=c$




#535634 CMR: $\frac{n+1}{n+2} - n.\frac{...

Gửi bởi le_hoang1995 trong 30-11-2014 - 21:41

 

Chứng minh bằng qui nạp với $n,k \in  \mathbb{N}*$

$\frac{n+1}{n+2} - n.\frac{\sum_{k=1}^{n}\frac{2^{k}}{k}}{2^{n+1}} < 0$
 
Giúp mình bài này với!!!

 

Bài này với $n=1$ thì BĐT sai, còn từ 2 trở đi mới đúng. Bạn xem lại đề nhé. Mình chứng minh như sau

Ta chứng minh bổ đề: với $n\geq2$ thì ta có BĐT

$$2+\frac{2^2}{2}+\frac{2^3}{3}+...+\frac{2^{n-1}}{n-1}\geq\frac{2^n}{n}$$

Với $n=2$ dễ thấy BĐT đúng

Giả sử BĐT đúng với $n=k$ suy ra $2+\frac{2^2}{2}+\frac{2^3}{3}+...+\frac{2^{k-1}}{k-1}\geq\frac{2^k}{k}$

Ta thấy $2+\frac{2^2}{2}+\frac{2^3}{3}+...+\frac{2^{k-1}}{k-1}+\frac{2^k}{k}\geq\frac{2^k}{k}+\frac{2^k}{k}=\frac{2^{k+1}}{k}>\frac{2^{k+1}}{k+1}$

Suy ra BĐT đúng với $n=k+1$

Theo nguyên lí quy nạp toán học, BĐT đúng với mọi $n\geq2$

________________________________________________

Trở lại BĐT ban đầu, ta có

$2+\frac{2^2}{2}+\frac{2^3}{3}+...+\frac{2^{n-1}}{n-1}\geq\frac{2^n}{n}$
$\Leftrightarrow 2+\frac{2^2}{2}+\frac{2^3}{3}+...+\frac{2^{n-1}}{n-1}+\frac{2^n}{n}\geq2.\frac{2^n}{n}=\frac{2^{n+1}}{n}$
$\Leftrightarrow n.\frac{\sum_{k=1}^{n}\frac{2^{k}}{k}}{2^{n+1}}\geq1$

Dễ thấy $\frac{n+1}{n+2}<1$

Vậy $\frac{n+1}{n+2}< n.\frac{\sum_{k=1}^{n}\frac{2^{k}}{k}}{2^{n+1}}$




#535385 Cho $a,b>0$, $a+b=2$ Tìm gtnn: P=$\frac...

Gửi bởi le_hoang1995 trong 29-11-2014 - 20:46

Mình sẽ cố gắng giải chi tiết

Áp dụng BĐT Bunhiacopxki, ta có

$\left ( a_1+a_2+...+a_n \right )\left ( \frac{b_1^2}{a_1}+\frac{b_2^2}{a_2}+...+\frac{b_n^n}{a_n} \right )\geq (b_1+b_2+...+b_n)^2$
$\Rightarrow \frac{b_1^2}{a_1}+\frac{b_2^2}{a_2}+...+\frac{b_n^n}{a_n}\geq \frac{(b_1+b_2+...+b_n)^2}{a_1+a_2+...+a_n}$

Áp dụng BĐT trên cho 10 phân số sau, ta có:

$3.\frac{1}{4a^2+2}+3.\frac{1}{4b^2+2}+4.\frac{1}{6ab}\geq \frac{(3.1+3.1+4.1)^2}{3.(4a^2+2)+3.(4b^2+2)+4.6ab}=\frac{100}{12.(a^2+2ab+b^2)+12}=\frac{5}{3}$

Và $ab\leq \left ( \frac{a+b}{2} \right )^2=1$

Suy ra ta có:

$3P=\frac{3}{4a^2+2}+\frac{3}{4b^2+2}+\frac{4}{6ab}+\frac{7}{3ab}\geq \frac{5}{3}+\frac{7}{3}=4$
$\Rightarrow P\geq \frac{4}{3}$

Đẳng thức xảy ra khi và chỉ khi
$\left\{\begin{matrix}
4a^2+2=4b^2+2=6ab\\a=b
\\ a+b=2

\end{matrix}\right.\Leftrightarrow a=b=1$




#535200 Chứng minh rằng: $\frac{2a}{b+c}+\frac...

Gửi bởi le_hoang1995 trong 28-11-2014 - 19:03

Cho a,b,c là các số thực dương. Chứng minh rằng: $\frac{2a}{b+c}+\frac{2b}{c+a}+\frac{2c}{a+b} \geq 3 + \frac{(a-b)^2+(b-c)^2+(c-a)^2}{(a+b+c)^2}$

BĐT tương đương với

$$\sum \left (  \frac{2a}{b+c}-1\right )\geq \sum \frac{(a-b)^2}{(a+b+c)^2}$$
$$\Leftrightarrow \sum \left ( \frac{a-b}{b+c}+\frac{a-c}{b+c} \right )\geq \sum \frac{(a-b)^2}{(a+b+c)^2}$$
$$\Leftrightarrow \sum (a-b).\left ( \frac{1}{b+c}-\frac{1}{a+c} \right )\geq \sum \frac{(a-b)^2}{(a+b+c)^2}$$
$$\Leftrightarrow \sum \frac{(a-b)^2}{(b+c)(a+c)}\geq \sum \frac{(a-b)^2}{(a+b+c)^2}$$

Dễ thấy

$$(a+c)(b+c)=c^2+ac+bc+ab<(a+b+c)^2$$
$$\Rightarrow \frac{(a-b)^2}{(a+c)(b+c)}\geq \frac{(a-b)^2}{(a+b+c)^2}$$
$$\Rightarrow  \sum \frac{(a-b)^2}{(b+c)(a+c)}\geq \sum \frac{(a-b)^2}{(a+b+c)^2}$$

Vậy BĐT đúng. Đẳng thức xảy ra khi $a=b=c$




#535180 CMR:$\left ( x^{2}+y^{2} \right )^{n...

Gửi bởi le_hoang1995 trong 28-11-2014 - 14:47

 

Chứng minh rằng :
$\left ( x^{2}+y^{2} \right )^{n} \geqslant 2^{n}x^{n}y^{n} + \left ( x^{n}-y^{n} \right )^{2}$
trong đó x,y là các số dương và n là số nguyên dương.  :namtay 

 

Chưa nghĩ ra cách trực tiếp nên sài tạm cách Quy nạp

Dễ thấy với $n=1$ hoặc $n=2$, BĐT đúng với mọi $x,y$ dương

Với $n\geq 3$ chia hai vế cho $y^{2n}$ rồi đặt ẩn, ta được BĐT $(a^2+1)^n\geq 2^n.a^n+(a^n-1)^2$        (1)

Với n=3, BĐT tương đương $a^4+a^2\geq2a^3$ đúng

----------------------------------------------------------

Ta chứng minh bổ đề $(a^2+1)(a^{n}-1)^2\geq (a^{n+1}-1)^2$ với mọi $n\geq3$         

Bổ đề tương đương

$(a^2+1).(a^n-1)^2\geq \left [ a(a^n-1)+(a-1) \right ]^2$
$\Leftrightarrow a^2.(a^n-1)^2+(a^n-1)^2\geq a^2.(a^n-1)^2+2a(a-1)(a^n-1)+(a-1)^2$
$\Leftrightarrow (a^n-1)^2-2.a(a-1).(a^n-1)+a^2(a-1)^2-a^2(a-1)^2-(a-1)^2\geq0$

$\Leftrightarrow \left [ (a^n-1)-a(a-1) \right ]^2-(a-1)^2(a^2+1)\geq 0$

$\Leftrightarrow (a-1)^n.(a^{n-1}+a^{n-2}+...+a^2+a+1-a)^2-(a-1)^2(a^2+1)\geq 0$

$\Leftrightarrow (a-1)^2.\left [ (a^{n-1}+a^{n-2}+...+a^2+1)^2-(a^2+1) \right ]\geq 0$

Vì $n\geq3$ nên $(a^{n-1}+a^{n-2}+...+a^2+1)^2-(a^2+1)\geq (a^2+1)^2-(a^2+1)=(a^2+1).a^2\geq 0$

Vậy bổ đề đúng với mọi $n\geq3$

-------------------------------------------------------------

Quay lại BĐT (1)

Giả sử BĐT đúng với mọi $n=m\geq3$

$$(a^2+1)^m\geq 2^m.a^m+(a^m-1)^2$$

Suy ra $$(a^2+1)^{m+1}=(a^2+1).(a^2+1)^m\geq (a^2+1).\left [ 2^m.a^m+(a^m-1)^2 \right ]$$

$$=(a^2+1).2^m.a^m+(a^2+1).(a^m-1)^2\geq 2a.2^m.a^m+(a^{m+1}-1)^2=2^{m+1}.a^{m+1}+(a^{m+1}-1)^2$$

Suy ra BĐT (1) cũng đúng với $n=m+1$, vậy BĐT đúng với mọi $n\geq3$

Đẳng thức xảy ra khi $a=1$ hay $x=y$

Tổng kết lại, BĐT đúng với mọi x, y dương và n nguyên dương




#535175 Chứng minh $2\sum a^{2014}\geq \sum a^{201...

Gửi bởi le_hoang1995 trong 28-11-2014 - 14:19

Cho $a; b; c$ là các số thực dương. Chứng minh:

$2(a^{2014}+b^{2014}+c^{2014})\geq a^{2013}(b+c)+b^{2013}(a+c)+c^{2013}(a+b)$

Áp dụng BĐT Cosi cho 2014 số, ta được

$$2013.a^{2014}+b^{2014}\geq 2014.\sqrt[2014]{a^{2013.2014}.b^{2014}}=2014.a^{2013}.b$$

Tương tự, ta được $$2013.a^{2014}+c^{2014}\geq 2014.\sqrt[2014]{a^{2013.2014}.c^{2014}}=2014.a^{2013}.c$$

Cộng lại ta được $$2.2013.a^{2014}+b^{2014}+c^{2014}\geq 2014.a^{2013}.(b+c)$$

Hoàn toàn tương tự, thêm hai BĐT nữa, cộng lại, ta được ĐPCM




#534238 Tìm GTLN, GTNN của hàm số: $y=sin^3x+cos^3x$

Gửi bởi le_hoang1995 trong 22-11-2014 - 19:36

Tìm GTLN, GTNN của hàm số: $y=sin^3x+cos^3x$

Theo tính chất lượng giác, ta có $sinx\leq 1$ và $cosx\leq 1$ với mọi x, suy ra

$sin^3x\leq sin^2x$ và   $ cos^3x\leq cos^2x$

Cộng hai vế, ta được $$sin^3x+cos^3x\leq sin^2x+cos^2x=1$$

Đẳng thức xảy ra khi $x=\frac{k\pi }{2}$




#534229 Tìm GTNN $P=\frac{a^{6}}{b+c}+\f...

Gửi bởi le_hoang1995 trong 22-11-2014 - 19:22

Cho a,b,c>0 và abc=1.Tìm GTNN $P=\frac{a^{6}}{b+c}+\frac{b^6}{c+a}+\frac{c^6}{a+b}$

Thêm 1 ý tưởng khác là dùng BĐT Trê-bư-sép và Nestbit

$$\frac{a^6}{b+c}+\frac{b^6}{c+a}+\frac{c^6}{a+b}\geq \frac{1}{3}.\left ( a^5+b^5+c^5 \right ).\left ( \frac{a}{b+c}+\frac{b}{c+a}+\frac{c}{a+b} \right )\geq \frac{1}{3}.3.\sqrt[3]{a^5.b^5.c^5}.\frac{3}{2}=\frac{3}{2}$$

Đẳng thức xảy ra khi và chỉ khi $a=b=c=1$




#534170 Tim GTNN $P=\frac{a^{2}\left ( b+c \right...

Gửi bởi le_hoang1995 trong 22-11-2014 - 12:37

Tính chất của tam giác vuông $a^2+b^2=c^2$ nên ta có

$$\frac{a^2(b+c)+b^2(a+c)}{abc}=\frac{ab(a+b)+c(a^2+b^2)}{abc}=\frac{a}{c}+\frac{b}{c}+\frac{c^2}{ab}$$ $$=\frac{a}{c}+\frac{b}{c}+\frac{c^2}{2\sqrt{2}ab}+\frac{(4-\sqrt{2}).c^2}{4ab}\geq 3.\sqrt[3]{\frac{ab.c^2}{c^2.2\sqrt{2}.ab}}+\frac{(4-\sqrt{2})(a^2+b^2)}{4ab}$$ $$=\frac{3\sqrt{2}}{2}+\frac{(4-\sqrt{2}).2ab}{4ab}=2+\sqrt{2}$$

Đẳng thức xảy ra tại các bắt đẳng thức Cô si trên, giải ra ta được $a=b=\frac{c}{\sqrt{2}}$ hay tam giác vuông cân




#359148 ÔN THI ĐẠI HỌC 2012

Gửi bởi le_hoang1995 trong 05-10-2012 - 18:37

Bài 30:
Ch0 các số thực $a,b$ và số thực dương $c$ thỏa mãn $a^2+b^2+ab=3c^2$.Chứng minh rằng:
$$a^3+b^3+4abc\leq 6c^3$$

Giả thiết tương đương $a^2c+b^2c+abc=3c^3$
$3c^2=(a+b)^2-ab\geq (a+b)^2-\frac{(a+b)^2}{4}=\frac{3(a+b)^2}{4}\Leftrightarrow 2c\geq a+b$
BĐT cần chứng minh tương đương với
$a^3+b^3+4abc\leq 2a^2c+2b^2c+2abc$
$\Leftrightarrow (a+b)(a^2+b^2-ab)+2abc\leq 2c(a^2+b^2)$
$\Leftrightarrow (a+b)(a^2+b^2-ab)-2c(a^2+b^2-ab)\leq 0$
$\Leftrightarrow (a^2+b^2-ab)(a+b-2c)\leq 0$
Đúng theo điều kiện. Dấu bằng xảy ra khi $a=b=c=1$
____________________________________________

Bài 31 Cho $0<y<x \leq 3$ và $x+y \leq 5$, tìm GTLN của các biểu thức
$S_2=x^2+y^2\\ S_3=x^3+y^3\\ S_n=x^n+y^n$


#358767 ÔN THI ĐẠI HỌC 2012

Gửi bởi le_hoang1995 trong 04-10-2012 - 13:46

Bài 27:Cho $x,y$ thực thỏa $1-y^2=x(x-y)$. Tìm Min,max của $$P=\frac{x^6+y^6-1}{x^3y+xy^3}$$
THPT Đồng Quan Hà Nội 2011

Điều kiện $x,y$ khác $0$. $1=x^2+y^2-xy \geq xy$. Không tồn tại $min(xy)$ vì khi cho $x\rightarrow 0\Rightarrow y\rightarrow -1\Rightarrow xy \to 0$. Dùng tam thức bậc hai thì chặn được $|x|\leq \frac{2}{\sqrt{3}}\Rightarrow |xy|< \frac{4}{3}$
Đặt $xy=t$ với điều kiện $t \in(\frac{-4}{3};1]$ Ta có $x^3y+xy^3=xy(x^2+y^2)=xy(1+xy)=t(1+t)$
$x^6+y^6-1=(x^2+y^2)^3-3x^2y^2(x^2+y^2)-1=(1+xy)^3-3x^2y^2(1+xy)-1=(1+t)^3-3t^2(1+t)-1=-2t^3+3t$
$\Rightarrow P=\frac{-2t^3+3t}{t(t+1)}=\frac{-2t^2+3}{t+1}$
$P'=\frac{-2t^2-4t-3}{(t+1)^2}<0$. Suy ra P nghịch biến
$\Rightarrow P(t)\geq P(1)=\frac{1}{2}$
Vậy $minP=\frac{1}{2}$ khi $t=1\Rightarrow x=y=1$
Không tồn tại maxP


#356802 $$(a^2b+b^2c+c^2a)(ab+bc+ca)\leq 9$$

Gửi bởi le_hoang1995 trong 26-09-2012 - 18:40

Cả ba bài đều dùng chung 1 ý tưởng như trên: giả sử $b$ là số nằm giữa $a$ và $c$, ta dễ chứng minh $a^2b+b^2c+c^2a\leq b(a^2+ac+c^2)$

Bài toán 2: Ch0 $a,b,c$ là các số thực không âm thỏa $a^2+b^2+c^2=3$.Chứng minh rằng:
$$a^2b+b^2c+c^2a-abc\leq 2$$

Suy ra $$a^2b+b^2c+c^2a-abc\leq b(a^2+ac+c^2)-abc=b(a^2+c^2)$$
$$=2\sqrt{b^2.\frac{a^2+c^2}{2}.\frac{a^2+c^2}{2}}\leq 2\sqrt{\left ( \frac{b^2+2.\frac{a^2+c^2}{2}}{3} \right )^3}=2$$

Bài toán 3: Ch0 $a,b,c$ là các số thực không âm thỏa $a+b+c=3$.Chứng minh rằng:
$$(a^2b+b^2c+c^2a)(ab+bc+ca)\leq 9$$

$$3VT\leq 3b.(a^2+ac+c^2).(ab+bc+ca)=3b.[(a+c)^2-ac].[ac+b(a+c)]$$
$$=3b.[(3-b)^2-ac].[ac+b(3-b)]\leq \left ( \frac{3b+9-6b+b^2-ac+ac+3b-b^2}{3} \right )^3=27$$
$$\Rightarrow VT \leq 9$$


#356675 Tìm min của P: $P=\frac{a^2}{c(a^2+c^2)}+...

Gửi bởi le_hoang1995 trong 25-09-2012 - 21:44

Cho a,b,c là các số thực thỏa mãn: $ab+bc+ca=3abc$

Tìm min của P:

$P=\frac{a^2}{c(a^2+c^2)}+\frac{b^2}{a(b^2+a^2)}+\frac{c^2}{b(c^2+b^2)}$

Đặt tương ứng $$(a;b;c)=(\frac{1}{x};\frac{1}{y};\frac{1}{z})$$
Suy ra giả thiết: $\frac{1}{xy}+\frac{1}{yz}+\frac{1}{zx}=\frac{3}{xyz}\Leftrightarrow x+y+z=3$
$$P=\sum \frac{\frac{1}{x^2}}{\frac{1}{z}.\left ( \frac{1}{x^2}+\frac{1}{z^2} \right )}=\sum \frac{\frac{1}{x^2}}{\frac{x^2+z^2}{x^2z^3}}=\sum \frac{z^3}{x^2+z^2}=\sum \left ( z-\frac{x^2z}{x^2+z^2} \right )$$
$$\geq \sum \left ( z-\frac{x^2z}{2xz} \right )=\sum \left ( z-\frac{x}{2} \right )=\frac{x+y+z}{2}=\frac{3}{2}$$
Dấu bằng xảy ra khi $a=b=c=1$